+1 Daumen
776 Aufrufe



ich beschäftige mich momentan mit Fastkörpern, genauer mit der Konstruktion nach Dickson, hierbei bin ich in der Literatur auf eine Frage gestoßen, die mit dem semi-direkten Produkt von Gruppen zusammenhängt, vielleicht kann mir jemand von euch da einen Tipp geben :)
Es geht um folgendes:

blob.png

Text erkannt:

Wir betrachten die affinen Abbildungen \( \varphi_{\left(q^{k}\right), a}: \mathbb{Z} /\left(q^{d}-1\right) \mathbb{Z}^{\rightarrow} \mathbb{Z} /\left(q^{d}-1\right) \mathbb{Z} \)
\( t \rightarrow q^{k} \cdot t+a, \) die eine Gruppe \( G \) der Ordnung d \( \left(q^{d}-1\right) \) bilden Es ist \( \pi: \mathbb{G} \rightarrow \mathbb{Z} / \mathrm{d} \mathbb{Z} \)
\( \varphi_{\left(q^{k}\right), a} \rightarrow k \)
ein Gruppenhomomorphi smus, außerdem gilt
G ist semi - direktes Produkt der Gruppen \( \mathbb{Z} /\left(\mathrm{q}^{\mathrm{d}}-1\right) \mathrm{z} \) und \( \mathbb{Z} / \mathrm{d} \mathbb{Z} \), wobe i
\( \mathbb{Z} / \mathrm{d} \mathbb{Z} \) wirkt via \( (\mathrm{k}, \mathrm{a}) \rightarrow \mathrm{q}^{\mathrm{k}} \cdot \mathrm{a} \)
Dieser letzte Punkt ist mir leider nicht klar, ich weiß dass man für das (äußere) semi - direkte Produkt einen Gruppenhomomorphi smus
\( \rho: \mathbb{Z} /\left(\mathrm{q}^{\mathrm{d}}-1\right) \mathrm{z}^{\rightarrow} \) Aut \( (\mathrm{Z} / \mathrm{d} \mathrm{z}) \) benötigt, mir ist aber leider nicht klar,
wie dieser in diesem Beispiel aussehen muss. Meine Überlegung war, dass Elemente aus \( \mathbb{Z} /\left(\mathrm{q}^{\mathrm{d}}-1\right) \mathrm{z} \) vermutlich auf die Operation \( \mathbb{Z} / \mathrm{d} \mathbb{Z} \times \mathbb{Z} / \mathrm{d} \mathrm{z} \rightarrow \mathbb{Z} / \mathrm{d} \mathbb{Z} \)
\( (\mathrm{k}, \mathrm{a}) \rightarrow \mathrm{q}^{\mathrm{k}} \cdot \) a abgebildet werden, aber weiter bin ich leider nicht gekommen.

 Wäre über jeden Tipp dankbar
LG

Avatar von

Sorry kleiner Fehler:

Die Operation muss natürlich von Z/dZxZ/q^d-1Z nach Z/q^d-1Z gehen!

1 Antwort

0 Daumen
 
Beste Antwort

Seien \( N, H \) Gruppen und \( \Theta: H \to \operatorname{Aut}(N) \) ein Homomorphismus. Wir betrachten das semidirekte Produkt \( N \rtimes_\Theta H \).

Dann wirkt doch \( H \) auf \( N \) via $$ H \times N \to N, (h,n) \mapsto \Theta(h)(n) $$

Jetzt gehen wir in dein Problem:

\( \mathbb{Z}/d \mathbb{Z} \) wirkt auf \( \mathbb{Z}/(q^d-1) \mathbb{Z} \) via \( (k,a) \mapsto q^ka \)

Wir identifizieren \( H = \mathbb{Z}/d \mathbb{Z} \), \( N = \mathbb{Z}/(q^d-1) \mathbb{Z} \), \( \Theta : \mathbb{Z}/d \mathbb{Z} \to \operatorname{Aut}\left( \mathbb{Z}/(q^d-1) \mathbb{Z}\right) \)

Wegen der gegebenen Wirkung ist \( \Theta(k)(a) = q^k a \), also

$$ \Theta : \mathbb{Z}/d \mathbb{Z} \to \operatorname{Aut}\left( \mathbb{Z}/(q^d-1)\mathbb{Z} \right), k \mapsto( a \mapsto q^k a ) $$

Die Abbildungen \( \varphi_{q^k,0} : a \mapsto q^k a \) sind Homomorphismen:

$$ \varphi_{q^k,0}(a+b) = q^k(a+b) = q^ka + q^kb = \varphi_{q^k,0}(a) + \varphi_{q^k,0}(b) $$

und bijektiv da \( q^k \in \left( \mathbb{Z}/(q^d-1)\mathbb{Z}  \right)^* \).

\( \Theta \) ist ein Homomorphismus: $$ \Theta(k_1 + k_2) = \varphi_{q^{k_1 + k_2},0} =  \varphi_{q^{k_1},0}  \circ \varphi_{q^{k_2},0}  = \Theta(k_1) \circ \Theta(k_2) $$

Avatar von 6,0 k

Jetzt kann man sich natürlich noch Fragen wie ein Isomorphismus \( \Psi: \mathbb{Z}/(q^d-1)\mathbb{Z} \rtimes_\Theta \mathbb{Z}/d\mathbb{Z} \cong G \) aussieht:

Für \( (a_1, k_1), (a_2, k_2) \in \mathbb{Z}/(q^d-1)\mathbb{Z} \rtimes_\Theta \mathbb{Z}/d\mathbb{Z} \) muss gelten

$$  \begin{aligned} \Psi( (\color{orange}{a_1 + q^{k_1}a_2}, \color{blue}{k_1+k_2})) &= \Psi( (a_1 + \Theta(k_1)(a_2), k_1 + k_2 ) ) \\ &= \Psi( (a_1, k_1) \cdot (a_2, k_2) ) \\ &= \Psi((\color{red}{a_1},\color{red}{ k_1})) \circ \Psi((\color{purple}{a_2}, \color{purple}{k_2})) \end{aligned} $$

Und wenn man sich mal die Verknüpfung zweier affine Abbildungen anschaut:

$$ \varphi_{q^{\color{red}{k_1}}, \color{red}{a_1}} \circ \varphi_{q^{\color{purple}{k_2}}, \color{purple}{a_2}} = \varphi_{q^{\color{blue}{k_1+k_2}}, \color{orange}{a_1 + q^{k_1}a_2}} $$

Sieht man schnell, dass \( \Psi( (a,k) ) = \varphi_{q^k,a} \) möglich ist.

Hallo Emnero,

 vielen Dank für deine hilfreiche Antwort!
Einen Frage habe ich allerdings noch: ist es dann nicht so, dass die Gruppe G nur einer Teilmenge des Semidirekten Produktes der beiden Gruppen entspricht ? Ich habe ja quasi im Semidirekten Produkt Elemente, die  nicht zu einer Abbildung der Form phi_q^k,a passen, oder?
LG und schönen Feiertag :)

Ich habe ja quasi im Semidirekten Produkt Elemente, die  nicht zu einer Abbildung der Form phi_qk,a passen, oder?

Welche Elemente hast du da so im Sinn? Denk da nochmal scharf drüber nach ;) Das \( \Psi \) ist ein Isomorphismus (= bijektiver Homomorphismus).

Dir auch einen schönen Feiertag!

P.S. Falls dir die Antwort geholfen hat, würde ich mich über einen Stern freuen :)

Ah natürlich, vielen Dank dir :)

Ein anderes Problem?

Stell deine Frage

Willkommen bei der Mathelounge! Stell deine Frage einfach und kostenlos

x
Made by a lovely community